Difference between revisions of "2006 AMC 10A Problems/Problem 2"
(→Solution) |
m (→Solution) |
||
Line 4: | Line 4: | ||
<math> \mathrm{(A) \ } -h\qquad \mathrm{(B) \ } 0\qquad \mathrm{(C) \ } h\qquad \mathrm{(D) \ } 2h\qquad \mathrm{(E) \ } h^3 </math> | <math> \mathrm{(A) \ } -h\qquad \mathrm{(B) \ } 0\qquad \mathrm{(C) \ } h\qquad \mathrm{(D) \ } 2h\qquad \mathrm{(E) \ } h^3 </math> | ||
== Solution == | == Solution == | ||
− | Plugging our values | + | Plugging our values into the defined [[operation]], we have |
− | <center><math>\displaystyle h^3-h</math></center> | + | <center><math>\displaystyle h\otimes h = h^3-h</math></center> |
− | Plugging in the function once more, we have | + | Plugging in the [[function]] once more, we have |
− | <center><math>\displaystyle h^3-(h^3-h)=h | + | <center><math>\displaystyle h\otimes(h\otimes h) = h\otimes (h^3 - h) = h^3-(h^3-h)=h</math></center> |
− | (C)</math> | + | |
+ | which is choice <math>(C)</math>. | ||
== See Also == | == See Also == | ||
*[[2006 AMC 10A Problems]] | *[[2006 AMC 10A Problems]] |